Đến nội dung

T M

T M

Đăng ký: 19-11-2011
Offline Đăng nhập: 09-07-2014 - 12:58
****-

#450573 $\lim_{x\to 0} \frac{x-sinx}{x^3...

Gửi bởi T M trong 15-09-2013 - 10:56

-_- không biết đúng không nhé : 

$lim \frac{1}{x^{2}}-lim\frac{1}{x^{2}}.lim\frac{sinx}{x}=lim\frac{1}{x^{2}}-lim\frac{1}{x^{2}}$

Vì $lim \frac{sinx}{x}=1$ mà , $x->0$ nhé

 

Chưa biết đúng sai như thế nào. Nhưng theo mình, bài của bạn 0 điểm.

 

Bạn mắc một lỗi rất cơ bản, $\lim(u+v)=\lim u + \lim v$, khi và chỉ khi $\lim u$ và $\lim v$ là HỮU HẠN. ( Xin lỗi vì mình không viết biến số tiến tới đâu, lười ý mà ^^)

 

Bài này để "tránh" dùng L'Hospital, có thể dùng chính định nghĩa đạo hàm, được viết trong SGK 11. Bạn nào chém nốt đi nhé :D




#450371 ĐỀ THI CHỌN ĐỘI TUYỂN TOÁN 12 TRƯỜNG THPT CHUYÊN PHAN NGỌC HIỂN

Gửi bởi T M trong 14-09-2013 - 22:09

Chuyên Phan Ngọc Hiển chuyên cho kiểu cấp số cộng, cấp số nhân thì phải :D




#448563 $$a^n+\frac{1}{a^n}-2 \geq n^2\l...

Gửi bởi T M trong 07-09-2013 - 21:08

Bước cuối có thể dùng BĐT Cauchy - Schwarz chắc nhàn hơn :P 

 

$$x^3+y^3 \geq \frac{\left ( x^2+y^2 \right )^2}{x+y} \geq \frac{\left ( \frac{(x+y)^2}{2} \right )^2}{x+y}=\frac{(x+y)^3}{4}$$

 

Cũng có thể chứng minh trực tiếp bằng Holder.

 

Chú ý là $x+y \geq 2$. 




#447216 $\frac{a^3}{(a+b)^3}+\frac{b^3}...

Gửi bởi T M trong 02-09-2013 - 12:43

1) Cho $a,b,c$ dương. CMR : $\frac{a^3}{(a+b)^3}+\frac{b^3}{(b+c)^3}+\frac{c^3}{(a+c)^3}\geq \frac{3}{8}$.

 

 

Lời giải:

 

$\bullet$ Trước tiên, thực hiện phép đổi biến $$\left ( \frac{b}{a} ; \frac{c}{b};\frac{a}{c} \right ) \to \left ( x;y;z \right ) \Rightarrow xyz=1$$

 

Viết lại điều phải chứng minh dưới dạng $$\sum \frac{1}{\left ( 1+a \right )^3} \geq \frac{3}{8}$$

 

$\bullet$ Sử dụng bất đẳng thức $AM-GM$ ta có đánh giá 

 

$$\frac{1}{(1+a)^3}+\frac{1}{(1+a)^3}+\frac{1}{8} \geq 3\sqrt[3]{\frac{1}{(1+a)^6.8}} =\frac{3}{2}.\frac{1}{(1+a)^2}$$

 

Điều phải chứng minh trở thành $$\sum \frac{1}{(1+x)^2} \geq \frac{3}{4}$$

 

Do tính đối xứng của BĐT, ta giả sử $xy \geq 1$.

 

Chứng minh bất đẳng thức phụ sau $\frac{1}{(1+x)^2}+\frac{1}{(1+y)^2} \geq \frac{1}{1+xy} \ \ \forall xy \geq 1$

 

Bài toán đưa về chứng minh $\frac{z^2+z+1}{(z+1)^2} \geq \frac{3}{4}$. Điều này đúng. 

Vậy ta có đpcm.




#446581 $\lim_ {x\to{1}}{\dfrac{x^...

Gửi bởi T M trong 31-08-2013 - 16:48

 

Tim: $\lim_ {x\to{1}}{\dfrac{x^{100}-2x+1}{x^{50}-2x+1}}$

Tim:  [TeX]\lim_ {x\to{1}}{\dfrac{x^{100}-2x+1}{x^{50}-2x+1}}[/TeX]

Lời giải: 

 

$$\lim_{x\to 1}\frac{x^{100}-2x+1}{x^{50}-2x+1}=\lim_{x\to 1}\frac{\left(x^{100}-1\right)-2\left(x-1\right)}{\left(x^{50}-1 \right)-2\left( x-1 \right) } = \lim_{x \to 1}\frac{x^{99}+x^{98}+........+1-2}{x^{49}+.......+1-2}=\frac{49}{24}$$




#446526 Tìm tất cả hàm $f$ thỏa mãn $ f:R \to R$ thỏa $...

Gửi bởi T M trong 31-08-2013 - 13:24

Bài 1. Tìm tất cả hàm $f$ thỏa mãn $ f:R \to R$ thỏa $$f(x+1)-f(x)=2^{-x}$$
 
Bài 2. Cho $a$ là một hằng số dương. Tìm $f$ xác định trên $R$ sao cho $$f(x+a)=-f(x)$$

 


  • LNH yêu thích


#442175 Tổng hợp các bài toán dãy số 30/4/2012

Gửi bởi T M trong 12-08-2013 - 12:43

Đây là tài liệu tổng hợp nhỏ của mình gồm hơn 20 bài toán dãy số lấy từ đề đề nghị của các trường năm 2012. Mọi người làm rồi chúng ta cùng thảo luận nhé :D

 

 

File gửi kèm




#441457 $a_{n+1}=a_n+\frac{1}{\sqrt{a_n...

Gửi bởi T M trong 09-08-2013 - 12:50

Cái mình thắc mắc là dự đoán số $\frac{3}{2}$ như thế nào ?




#437546 TUYỂN TẬP CÁC BÀI TOÁN VỀ ELIP - LUYỆN THI ĐẠI HỌC

Gửi bởi T M trong 23-07-2013 - 18:49

Bài viết hay quá ạ :) Latex rất đẹp nữa :)




#436899 $a_{n+1}=a_n+\frac{1}{\sqrt{a_n...

Gửi bởi T M trong 21-07-2013 - 17:22

Đề bài: Dãy số $a_n$ được xác định bởi $a_1=1$ và $a_{n+1}=a_n+\frac{1}{\sqrt{a_n}}$.
 
Hãy tìm tất cả các số thực $\beta$ để dãy $\frac{a_n^\beta}{n}$ có giới hạn hữu hạn khác $0$
 
VN TST 1993

 




#435848 Chứng minh định lí $Stolz$

Gửi bởi T M trong 17-07-2013 - 18:56

Bài toán (Định lí $Stolz$). Nếu 2 dãy số $x_n$ và $y_n$ thỏa mãn đồng thời

  • $y_{n+1}>y_n$ với mọi $n \in N*$
  • $y_n \to +\infty$
  • $\lim\left ( \frac{x_n-x_{n-1}}{y_n-y_{n-1}} \right )=A$

Chứng minh rằng $\lim\left ( \frac{x_n}{y_n} \right )=A$




#435272 $\sum \frac{a^2}{\sqrt{2a^2 +ab +b^2...

Gửi bởi T M trong 14-07-2013 - 19:21

Cho a,b,c là các số thực dương.
Chứng minh rằng:

\[\dfrac{{{a^2}}}{{\sqrt {2{a^2} + ab + {b^2}} }} + \dfrac{{{b^2}}}{{\sqrt {2{b^2} + bc + {c^2}} }} + \dfrac{{{c^2}}}{{\sqrt {2{c^2} + ca + {a^2}} }} \ge \dfrac{{a + b + c}}{2}\]

 

Bạn xem thêm ở tập tin đính kèm về phương pháp này nhé :)

File gửi kèm




#435271 Cho$x,y,z>0$ thỏa $x^3 + y^3 + z^3=3$. CMR: $...

Gửi bởi T M trong 14-07-2013 - 19:17

Cho $x,y,z>0$ thỏa $x^3 + y^3 + z^3=3$. Chứng minh rằng:

$$\dfrac{xy}{z}+\dfrac{xz}{y} + \dfrac{yz}{x} \geq 3.$$

 

Xem thêm ở đây.




#435110 $P\geqslant \frac{5}{3}$

Gửi bởi T M trong 13-07-2013 - 23:06

Cho $x;y;z> 0$.Chứng minh rằng :

$P=\frac{2xy}{(z+x)(z+y)}+\frac{2yz}{(x+y)(x+z)}+\frac{3xz}{(y+z)(y+x)}\geqslant \frac{5}{3}.$

 

(trích đề thi học sinh giỏi lớp 11-Quảng Bình 2011)

--------------------------------

CBBX

 

Hướng giải:

 

Bằng khai triển trực tiếp ta đưa bất đẳng thức cần chứng minh thành

 

$$xy(x+y)+yz(y+z)+4xz\left ( x+z \right )\geq 10xyz$$

 

Điều này tương đương với

 

$$\frac{x+y}{z}+\frac{y+z}{x}+\frac{4(x+z)}{y}\geq 10$$

 

Áp dụng $AM-GM$ từng cặp là ra.




#432776 [TSĐH 2013] Đề thi môn toán khối A, A1

Gửi bởi T M trong 04-07-2013 - 12:22

Số phức làm đơn giản như thế này cũng được :P

Áp dụng công thức Morvie ta có

 

$z=2\left ( \frac{1}{2}+\frac{\sqrt{3}}{2}.i \right )=2\left ( \cos{\frac{\pi}{3}+\sin{\frac{\pi}{3}i}} \right ) \Longrightarrow z^5=2^5\left ( \frac{1}{2}-\frac{\sqrt{3}}{2}i \right ) \\ \Longrightarrow w=i\left ( 1-\sqrt{3} \right )16+16\left (1+\sqrt{3} \right )$

 

Xong rồi :P